LSAT and Law School Admissions Forum

Get expert LSAT preparation and law school admissions advice from PowerScore Test Preparation.

User avatar
 Dave Killoran
PowerScore Staff
  • PowerScore Staff
  • Posts: 5850
  • Joined: Mar 25, 2011
|
#85629
Complete Question Explanation

(The complete setup for this game can be found here: viewtopic.php?f=386&t=3729)

The correct answer choice is (A)

The condition in the question stem forces the objects into Template 1, with K washed fifth:
Capture4.PNG
Accordingly, answer choice (A) cannot be true and is correct. By the way, Template 2 would not be
applicable because K would have to be third, which is impossible due to the third rule.

Get the most out of your LSAT Prep Plus subscription.

Analyze and track your performance with our Testing and Analytics Package.